Difference between revisions of "2017 AMC 10A Problems/Problem 1"

(Video Solution)
(10 intermediate revisions by 7 users not shown)
Line 1: Line 1:
==Problem==
+
== Problem ==
  
 
What is the value of <math>(2(2(2(2(2(2+1)+1)+1)+1)+1)+1)</math>?
 
What is the value of <math>(2(2(2(2(2(2+1)+1)+1)+1)+1)+1)</math>?
Line 6: Line 6:
  
  
==Solution 1==
+
== Solution 1 ==
  
 
Notice this is the term <math>a_6</math> in a recursive sequence, defined recursively as <math>a_1 = 3, a_n = 2a_{n-1} + 1.</math> Thus:
 
Notice this is the term <math>a_6</math> in a recursive sequence, defined recursively as <math>a_1 = 3, a_n = 2a_{n-1} + 1.</math> Thus:
 
<cmath>\begin{split}
 
<cmath>\begin{split}
a_2 = 3*2 + 1 = 7.\\
+
a_2 = 3 \cdot 2 + 1 = 7.\\
a_3 = 7 *2 + 1 = 15.\\
+
a_3 = 7 \cdot 2 + 1 = 15.\\
a_4 = 15*2 + 1 = 31.\\
+
a_4 = 15 \cdot 2 + 1 = 31.\\
a_5 = 31*2 + 1 = 63.\\
+
a_5 = 31 \cdot 2 + 1 = 63.\\
a_6 = 63*2 + 1 = \boxed{\textbf{(C)}\ 127}
+
a_6 = 63 \cdot 2 + 1 = \boxed{\textbf{(C)}\ 127}
 
\end{split}</cmath>
 
\end{split}</cmath>
  
==Solution 2==
+
Minor LaTeX edits by fasterthanlight
 +
 
 +
== Solution 2 ==
 
Starting to compute the inner expressions, we see the results are <math>1, 3, 7, 15, \ldots</math>. This is always <math>1</math> less than a power of <math>2</math>. The only admissible answer choice by this rule is thus <math>\boxed{\textbf{(C)}\ 127}</math>.
 
Starting to compute the inner expressions, we see the results are <math>1, 3, 7, 15, \ldots</math>. This is always <math>1</math> less than a power of <math>2</math>. The only admissible answer choice by this rule is thus <math>\boxed{\textbf{(C)}\ 127}</math>.
  
==Solution 3==
+
== Solution 3 ==
  
 
Working our way from the innermost parenthesis outwards and directly computing, we have <math>\boxed{\textbf{(C) } 127}</math>.
 
Working our way from the innermost parenthesis outwards and directly computing, we have <math>\boxed{\textbf{(C) } 127}</math>.
  
==Solution 4==
+
== Solution 4 ==
  
 
If you distribute this you get a sum of the powers of <math>2</math>.  The largest power of <math>2</math> in the series is <math>64</math>, so the sum is <math>\boxed{\textbf{(C)}\ 127}</math>.
 
If you distribute this you get a sum of the powers of <math>2</math>.  The largest power of <math>2</math> in the series is <math>64</math>, so the sum is <math>\boxed{\textbf{(C)}\ 127}</math>.
  
==Solution 5: Guessing Strategies for Lazy People==
+
==Video Solution==
"I'm too lazy, so I'll try to guess this," says the lazy person. Well then this is the solution for you! (note that in general however, these guessing strategies are just meant to be confidence boosters for your numerical thinking). If you look up a probability table for each of the answer choices on the AMC, you will get that <math>\boxed{\text{C}}</math>.
+
https://youtu.be/str7kmcRMY8
 +
 
 +
https://youtu.be/kA6W8SwjitA
 +
 
 +
~savannahsolver
 +
 
 +
== See Also ==
 +
 
 
{{AMC10 box|year=2017|ab=A|before=First Problem|num-a=2}}
 
{{AMC10 box|year=2017|ab=A|before=First Problem|num-a=2}}
 
{{MAA Notice}}
 
{{MAA Notice}}

Revision as of 17:09, 16 June 2020

Problem

What is the value of $(2(2(2(2(2(2+1)+1)+1)+1)+1)+1)$?

$\textbf{(A)}\ 70\qquad\textbf{(B)}\ 97\qquad\textbf{(C)}\ 127\qquad\textbf{(D)}\ 159\qquad\textbf{(E)}\ 729$


Solution 1

Notice this is the term $a_6$ in a recursive sequence, defined recursively as $a_1 = 3, a_n = 2a_{n-1} + 1.$ Thus: \[\begin{split} a_2 = 3 \cdot 2 + 1 = 7.\\ a_3 = 7 \cdot 2 + 1 = 15.\\ a_4 = 15 \cdot 2 + 1 = 31.\\ a_5 = 31 \cdot 2 + 1 = 63.\\ a_6 = 63 \cdot 2 + 1 = \boxed{\textbf{(C)}\ 127} \end{split}\]

Minor LaTeX edits by fasterthanlight

Solution 2

Starting to compute the inner expressions, we see the results are $1, 3, 7, 15, \ldots$. This is always $1$ less than a power of $2$. The only admissible answer choice by this rule is thus $\boxed{\textbf{(C)}\ 127}$.

Solution 3

Working our way from the innermost parenthesis outwards and directly computing, we have $\boxed{\textbf{(C) } 127}$.

Solution 4

If you distribute this you get a sum of the powers of $2$. The largest power of $2$ in the series is $64$, so the sum is $\boxed{\textbf{(C)}\ 127}$.

Video Solution

https://youtu.be/str7kmcRMY8

https://youtu.be/kA6W8SwjitA

~savannahsolver

See Also

2017 AMC 10A (ProblemsAnswer KeyResources)
Preceded by
First Problem
Followed by
Problem 2
1 2 3 4 5 6 7 8 9 10 11 12 13 14 15 16 17 18 19 20 21 22 23 24 25
All AMC 10 Problems and Solutions

The problems on this page are copyrighted by the Mathematical Association of America's American Mathematics Competitions. AMC logo.png